ImageVerifierCode 换一换
格式:DOC , 页数:11 ,大小:250.50KB ,
资源ID:1396392      下载积分:2000 积分
快捷下载
登录下载
邮箱/手机:
温馨提示:
如需开发票,请勿充值!快捷下载时,用户名和密码都是您填写的邮箱或者手机号,方便查询和重复下载(系统自动生成)。
如填写123,账号就是123,密码也是123。
特别说明:
请自助下载,系统不会自动发送文件的哦; 如果您已付费,想二次下载,请登录后访问:我的下载记录
支付方式: 支付宝扫码支付 微信扫码支付   
注意:如需开发票,请勿充值!
验证码:   换一换

加入VIP,免费下载
 

温馨提示:由于个人手机设置不同,如果发现不能下载,请复制以下地址【http://www.mydoc123.com/d-1396392.html】到电脑端继续下载(重复下载不扣费)。

已注册用户请登录:
账号:
密码:
验证码:   换一换
  忘记密码?
三方登录: 微信登录  

下载须知

1: 本站所有资源如无特殊说明,都需要本地电脑安装OFFICE2007和PDF阅读器。
2: 试题试卷类文档,如果标题没有明确说明有答案则都视为没有答案,请知晓。
3: 文件的所有权益归上传用户所有。
4. 未经权益所有人同意不得将文件中的内容挪作商业或盈利用途。
5. 本站仅提供交流平台,并不能对任何下载内容负责。
6. 下载文件中如有侵权或不适当内容,请与我们联系,我们立即纠正。
7. 本站不保证下载资源的准确性、安全性和完整性, 同时也不承担用户因使用这些下载资源对自己和他人造成任何形式的伤害或损失。

版权提示 | 免责声明

本文(【考研类试卷】考研数学二(线性方程组)模拟试卷26及答案解析.doc)为本站会员(rimleave225)主动上传,麦多课文库仅提供信息存储空间,仅对用户上传内容的表现方式做保护处理,对上载内容本身不做任何修改或编辑。 若此文所含内容侵犯了您的版权或隐私,请立即通知麦多课文库(发送邮件至master@mydoc123.com或直接QQ联系客服),我们立即给予删除!

【考研类试卷】考研数学二(线性方程组)模拟试卷26及答案解析.doc

1、考研数学二(线性方程组)模拟试卷 26及答案解析(总分:72.00,做题时间:90 分钟)一、选择题(总题数:13,分数:26.00)1.选择题下列每题给出的四个选项中,只有一个选项符合题目要求。(分数:2.00)_2.设 (分数:2.00)A.t=6时,必有秩(P)=1B.t=6时,必有秩(P)=2C.t6 时,必有秩(P)=1D.t6 时,必有秩(P)=23.设非齐次线性方程组 Ax=b有两个不同解 1 和 2 ,其导出组的一个基础解系为 1 , 2 ,c 1 ,c 2 为任意常数,则方程组 Ax=b的通解为(分数:2.00)A.c 1 1 +c 2 ( 1 + 2 )+ B.c 1 1

2、+c 2 ( 1 - 2 )+ C.c 1 1 +c 2 ( 1 + 2 )+ D.c 1 1 +c 2 ( 1 - 2 )+ 4.设 1 =(1,0,2) T 及 2 =(0,1,-1) T 都是线性方程组 Ax=0的解,则其系数矩阵 A= (分数:2.00)A.B.C.D.5.设 A为 mn矩阵,则齐次线性方程组 Ax=0仅有零解的充要条件是 A的(分数:2.00)A.列向量组线性无关B.列向量组线性相关C.行向量组线性无关D.行向量组线性相关6.设齐次线性方程组 (分数:2.00)A.=-2 且B=0B.=-2 且B0C.=1 且B=0D.=1 且B07.设矩阵 A mn 的秩为 r(A

3、)=mn,b 为任一 m维列向量,则(分数:2.00)A.线性方程组 Ax=b必无解B.线性方程组 Ax=b必有唯一解C.线性方程组 Ax=b必有无穷多解D.A的任意 m个列向量都线性无关8.设矩阵 A mn 的秩为 r,对于非齐次线性方程组 AX=b,(分数:2.00)A.当 r=m时,Ax=b 必有解B.当 r=n时,Ax=b 必有唯一解C.当 m=n时,Ax=b 必有唯一解D.当 rn 时,Ax=b 必有无穷多解9.设 1 , 2 , 3 是 4元非齐次线性方程组 Ax=b的 3个解向量,且秩(A)=3, 1 =(1,2,3,4) T , 2 + 3 =(0,1,2,3) T ,c 表示

4、任意常数,则线性方程组 Ax=b的通解 x= (分数:2.00)A.B.C.D.10.设 A为 n阶实矩阵,A T 是 A的转置矩阵,则对于线性方程组():Ax=0 和():A T Ax=0,必有(分数:2.00)A.()的解是()的解,()的解也是()的解B.()的解是()的解,但()的解不是()的解C.()的解不是()的解,()的解也不是()的解D.()的解是()的解,但()的解不是()的解11.设有齐次线性方程组 Ax=0和 Bx=0,其中 A、B 均为 mn矩阵,现有 4个命题: 若 Ax=0的解均是Bx=0的解,则秩(A)秩(B); 若秩(A)秩(B),则 Ax=0的解均是 Bx=0

5、的解; 若 Ax=0与 Bx=0同解,则秩(A)=秩(B); 若秩(A)=秩(B),则 Ax=0与 Bx=0同解 以上命题中正确的是(分数:2.00)A.B.C.D.12.设 A是,2 阶矩阵, 是 n维列向量,且秩 (分数:2.00)A.Ax= 必有无穷多解B.Ax= 必有唯一解C.=0仅有零解D.=0必有非零解13.设 n阶矩阵 A的伴随矩阵 A * O,若 1 , 2 , 3 , 4 是非齐次线性方程组 Ax=b的互不相等的解,则对应的齐次线性方程组 Ax=0的基础解系(分数:2.00)A.不存在B.仅含一个非零解向量C.含有两个线性无关的解向量D.含有三个线性无关的解向量二、填空题(总

6、题数:5,分数:10.00)14.设 (分数:2.00)填空项 1:_15.若方程组 (分数:2.00)填空项 1:_16.若 3阶非零方阵 B的每一列都是方程组 (分数:2.00)填空项 1:_填空项 1:_17.设 n阶方阵 A的各行元素之和均为零,且秩(A)=n-1,则齐次线性方程组 AX=0的通解为 1.(分数:2.00)填空项 1:_18.已知线性方程组 (分数:2.00)填空项 1:_三、解答题(总题数:18,分数:36.00)19.解答题解答应写出文字说明、证明过程或演算步骤。(分数:2.00)_20.设向量 1 =(1,0,2,3), 2 =(1,1,3,5), 3 =(1,-

7、1,a+2,1), 4 =(1,2,4,a+8),=(1,1,b+3,5)问:a,b 为何值时, 不能用 1 , 2 , 3 , 4 线性表示;a,b 为何值时, 能用 1 , 2 , 3 , 4 线性表示,并写出该表达式(分数:2.00)_21.问 a、b 为何值时,线性方程组 (分数:2.00)_22. 为何值时,线性方程组 (分数:2.00)_23.设 4元线性方程组()为 (分数:2.00)_24.已知线性方程组 的一个基础解系为:(b 11 ,b 12 ,b 1,2n ) T ,(b 21 ,b 22 ,b 2,2n ) T ,(b n1 ,b n2 ,b n,2n ) T 试写出线

8、性方程组 (分数:2.00)_25.设 1 , 2 , s 为线性方程组 Ax=0的一个基础解系, 1 =t 1 1 +t 2 2 , 2 =t 1 2 +t 2 3 , s =t 1 1 +t 2 1 ,其中 t 1 ,t 2 为实常数试问 t 1 ,t 2 满足什么关系时, 1 , 2 , m 也为 AX=0的一个基础解系.(分数:2.00)_26.设有 3维列向量 (分数:2.00)_27.已知线性方程组 (分数:2.00)_28.k为何值时,线性方程组 (分数:2.00)_29.设有线性方程组 (分数:2.00)_30.设矩阵 A、B 的行数都是 m证明:矩阵方程 AX=B有解的充分必

9、要条件是 r(A)=r(AB)(分数:2.00)_31.设矩阵 (分数:2.00)_32.已知齐次线性方程组 其中 (分数:2.00)_33.设 A为 n阶方阵(n2),A * 为 A的伴随矩阵,证明: (分数:2.00)_34.设 1 =(1,2,0) T , 2 =(1,a+2,-3a) T , 3 =(-1,-b-2,a+2b) T ,=(1,3,-3) T ,试讨论当 a,b 为何值时, () 不能由 1 , 2 , 3 线性表示; () 可由 1 , 2 , 3 惟一地线性表示,并求出表示式; () 可由 1 , 2 , 3 线性表示,但表示式不惟一,并求表示式(分数:2.00)_3

10、5.已知(1,-1,1,-1) T 是线性方程组 (分数:2.00)_36.已知齐次线性方程组 (分数:2.00)_考研数学二(线性方程组)模拟试卷 26答案解析(总分:72.00,做题时间:90 分钟)一、选择题(总题数:13,分数:26.00)1.选择题下列每题给出的四个选项中,只有一个选项符合题目要求。(分数:2.00)_解析:2.设 (分数:2.00)A.t=6时,必有秩(P)=1B.t=6时,必有秩(P)=2C.t6 时,必有秩(P)=1 D.t6 时,必有秩(P)=2解析:解析:当 t6 时,秩(Q)=2,且由 PQ=0知 Q的每一列都是方程组 PX=0的解,故 PX=0至少有 2

11、个线性无关的解, 基础解系所含向量个数 3-秩(P)2,3.设非齐次线性方程组 Ax=b有两个不同解 1 和 2 ,其导出组的一个基础解系为 1 , 2 ,c 1 ,c 2 为任意常数,则方程组 Ax=b的通解为(分数:2.00)A.c 1 1 +c 2 ( 1 + 2 )+ B.c 1 1 +c 2 ( 1 - 2 )+ C.c 1 1 +c 2 ( 1 + 2 )+ D.c 1 1 +c 2 ( 1 - 2 )+ 解析:解析:因 1 , 1 - 2 是与基础解系 1 , 2 等价的线性无关向量组,故 1 , 1 - 2 也是 Ax=0的基础解系,又由 (A 1 +A 2 )= (B+B)=

12、b知 4.设 1 =(1,0,2) T 及 2 =(0,1,-1) T 都是线性方程组 Ax=0的解,则其系数矩阵 A= (分数:2.00)A. B.C.D.解析:解析:由条件知 Ax=0至少有两个线性无关解,因此其基础解系所含向量个数至少为 2,即 3-r(A)2,5.设 A为 mn矩阵,则齐次线性方程组 Ax=0仅有零解的充要条件是 A的(分数:2.00)A.列向量组线性无关 B.列向量组线性相关C.行向量组线性无关D.行向量组线性相关解析:解析:设 A按列分块为 A= 1 , 2 , n ,则方程组 Ax=0的向量形式是 x 1 1 +x 2 2 +x n n =0,由此可知 Ax=0仅

13、有零解 x 1 1 +x 2 2 +x n n =0,仅在 x 1 =x 2 =x n =0时成立 6.设齐次线性方程组 (分数:2.00)A.=-2 且B=0B.=-2 且B0C.=1 且B=0 D.=1 且B0解析:7.设矩阵 A mn 的秩为 r(A)=mn,b 为任一 m维列向量,则(分数:2.00)A.线性方程组 Ax=b必无解B.线性方程组 Ax=b必有唯一解C.线性方程组 Ax=b必有无穷多解 D.A的任意 m个列向量都线性无关解析:解析:注意增广矩阵只有 m行,其秩不会大于 m,故由 m=r(A)tAbm,8.设矩阵 A mn 的秩为 r,对于非齐次线性方程组 AX=b,(分数

14、:2.00)A.当 r=m时,Ax=b 必有解 B.当 r=n时,Ax=b 必有唯一解C.当 m=n时,Ax=b 必有唯一解D.当 rn 时,Ax=b 必有无穷多解解析:9.设 1 , 2 , 3 是 4元非齐次线性方程组 Ax=b的 3个解向量,且秩(A)=3, 1 =(1,2,3,4) T , 2 + 3 =(0,1,2,3) T ,c 表示任意常数,则线性方程组 Ax=b的通解 x= (分数:2.00)A.B.C. D.解析:解析:由 Ax=b的解的结构知关键在于求出 Ax=0的基础解系,由于 Ax=0的基础解系所含解向量个数为 4-秩(A)=4-3=1,因此 Ax=0的任意一个非零解都

15、可作为 Ax=0的基础解系易知 =2 1 -( 2 + 3 )=(2,3,4,5) T 是 Ax=0的一个非零解,故 可作为 Ax=0的基础解系,所以,Ax=b 的通解为x= 1 +c只有选项(C)正确10.设 A为 n阶实矩阵,A T 是 A的转置矩阵,则对于线性方程组():Ax=0 和():A T Ax=0,必有(分数:2.00)A.()的解是()的解,()的解也是()的解 B.()的解是()的解,但()的解不是()的解C.()的解不是()的解,()的解也不是()的解D.()的解是()的解,但()的解不是()的解解析:解析:若 x满足 Ax=0,两端左乘 A T ,得 A T x=0,故

16、Ax=0的解都是 A T Ax=0的解;若 x满足 A T Ax=0,两端左乘 x T ,得(x T A T )(Ax)=0,即(Ax) T (Ax)=0,或Ax 2 =0,得 Ax=0,所以 A T Ax=0的解也都是 Ax=0的解因此()与()同解,只有选项(A)正确.11.设有齐次线性方程组 Ax=0和 Bx=0,其中 A、B 均为 mn矩阵,现有 4个命题: 若 Ax=0的解均是Bx=0的解,则秩(A)秩(B); 若秩(A)秩(B),则 Ax=0的解均是 Bx=0的解; 若 Ax=0与 Bx=0同解,则秩(A)=秩(B); 若秩(A)=秩(B),则 Ax=0与 Bx=0同解 以上命题中

17、正确的是(分数:2.00)A.B. C.D.解析:解析:若 Ax=0的解均是 Bx=0的解,则 Ax=0的解空间是 Bx=0的解空间的子空间,从而有 n-r(A)n-r(B),12.设 A是,2 阶矩阵, 是 n维列向量,且秩 (分数:2.00)A.Ax= 必有无穷多解B.Ax= 必有唯一解C.=0仅有零解D.=0必有非零解 解析:解析:注意选项(D)中的方程组是 n+1元方程组,而其系统矩阵的秩等于 A nn 的秩,它最大是n,必小于 n+1,因而该齐次线性方程组必有非零解13.设 n阶矩阵 A的伴随矩阵 A * O,若 1 , 2 , 3 , 4 是非齐次线性方程组 Ax=b的互不相等的解

18、,则对应的齐次线性方程组 Ax=0的基础解系(分数:2.00)A.不存在B.仅含一个非零解向量 C.含有两个线性无关的解向量D.含有三个线性无关的解向量解析:解析:由 A * O 知 A * 至少有一个元素 A ij =(-1) i+j M ij 0,故 A的余子式 M ij 0,而 M ij 为 A的 n-1阶子式,故 r(A)n-1,又由 Ax=b有解且不唯一知 r(A)n,故 r(A)=n-1因此,Ax=0 的基础解系所含向量个数为 n-r(A)=n-(n-1)=1,只有(B)正确二、填空题(总题数:5,分数:10.00)14.设 (分数:2.00)填空项 1:_ (正确答案:正确答案:

19、(1,0,0,0,0) T)解析:解析:由于方程组的系数行列式A T =A= 15.若方程组 (分数:2.00)填空项 1:_ (正确答案:正确答案:a 1 +a 2 +a 3 +a 4 =0)解析:解析:对方程组的增广矩阵施行初等行变换: 由阶梯形矩阵 B及方程组 Ax=b有解判定定理知,方程组有解 16.若 3阶非零方阵 B的每一列都是方程组 (分数:2.00)填空项 1:_ (正确答案:正确答案:1)填空项 1:_ (正确答案:0)解析:解析:由条件知方程组有非零解,故其系数行列式 A= 17.设 n阶方阵 A的各行元素之和均为零,且秩(A)=n-1,则齐次线性方程组 AX=0的通解为

20、1.(分数:2.00)填空项 1:_ (正确答案:正确答案:x=k,其中 k作为任意常数)解析:解析:n 元齐次线性方程组 Ax=0的基础解系所含向量个数为 n-r(A)=n-(n-1)=1,故 Ax=0的任一非零解都可作为它的基础解系由 A=(a ij ) nn 的元素满足 18.已知线性方程组 (分数:2.00)填空项 1:_ (正确答案:正确答案:-1)解析:解析:A ,当 a3 且 a-1 时有唯一解;当 a=3时,秩(A)=秩 =23,有无穷多解;当 a=-1时,秩(A)=2,秩三、解答题(总题数:18,分数:36.00)19.解答题解答应写出文字说明、证明过程或演算步骤。(分数:2

21、.00)_解析:20.设向量 1 =(1,0,2,3), 2 =(1,1,3,5), 3 =(1,-1,a+2,1), 4 =(1,2,4,a+8),=(1,1,b+3,5)问:a,b 为何值时, 不能用 1 , 2 , 3 , 4 线性表示;a,b 为何值时, 能用 1 , 2 , 3 , 4 线性表示,并写出该表达式(分数:2.00)_正确答案:(正确答案:当 a=-1,b0 时, 不能用 1 , 2 , 3 , 4 线性表示; 当 a-1时,有唯一的线性表示: = )解析:21.问 a、b 为何值时,线性方程组 (分数:2.00)_正确答案:(正确答案:当 a1 时有唯一解;当 a=1且

22、 b-1 时,无解;当 a=1且 b=-1时,通解为 x 1 =-1+c 1 +c 2 ,x 2 =1-2x 1 -2x 2 ,x 1 =c 2 ,x 4 =c 2 (c 1 ,c 2 为任意常数)或 )解析:22. 为何值时,线性方程组 (分数:2.00)_正确答案:(正确答案:当 1 时无解;当 =1 时,通解为 x 1 =1-c,x 2 =-1+2c,x 3 =c.(c为任意常数)解析:23.设 4元线性方程组()为 (分数:2.00)_正确答案:(正确答案:(1)由系数矩阵的初等行变换:A= (x 3 ,x 4 任意),令 x 3 =1,x 4 =0,得 1 =(0,0,1,0) T

23、;令 x 3 =0,x 4 =1,得 2 =(-1,1,0,1) T ,则 1 , 2 就是()的一个基础解系 (2)若 x是()和()的公共解,则存在常数 1 , 2 , 3 , 4 ,使 由此得 1 , 2 , 3 , 4 满足齐次线性方程组 )解析:24.已知线性方程组 的一个基础解系为:(b 11 ,b 12 ,b 1,2n ) T ,(b 21 ,b 22 ,b 2,2n ) T ,(b n1 ,b n2 ,b n,2n ) T 试写出线性方程组 (分数:2.00)_正确答案:(正确答案:记方程组()、()的系数矩阵分别为 A、B,则可以看出题给的()的基础解系中的 n个向量就是 B

24、的 n个行向量的转置向量因此,由()的基础解系可知 AB T =O 转置即得 BA T =O 因此可知 A T 的 n个列向量即 A的 n个行向量的转置向量都是方程组()的解向量 由于 B的秩为n(B的行向量组线性无关),故()的解空间的维数为 2n-r(B)=2n-n=n,所以()的任何 n个线性无关的解就是()的一个基础解系已知()的基础解系含 n个向量,即 2n-r(A)=n,故 r(A)=n,于是可知 A的 n个行向量线性无关,从而它们的转置向量构成()的一个基础解系,因此()的通解为 y=c 1 (a 11 ,a 12 ,a 1,2n ) T +c 2 (a 21 ,a 22 ,a

25、2,2n ) T +c(a n1 ,a n2 ,a n,2n ) T 其中 c 1 ,c 2 ,c n 为任意常数)解析:25.设 1 , 2 , s 为线性方程组 Ax=0的一个基础解系, 1 =t 1 1 +t 2 2 , 2 =t 1 2 +t 2 3 , s =t 1 1 +t 2 1 ,其中 t 1 ,t 2 为实常数试问 t 1 ,t 2 满足什么关系时, 1 , 2 , m 也为 AX=0的一个基础解系.(分数:2.00)_正确答案:(正确答案:由 Ax=0的解的线性组合都是解知, 1 , 2 , s 都是 Ax=0的解向量由于已知 Ax=0的基础解系含 s个向量,所以,只要 1

26、 , 2 , s 线性无关就可作为基础解系,否则不能作为基础解系由于 1 , 2 , s 由线性无关向量组 1 , 2 , s 线性表示的系数矩阵为 s阶方阵 故 1 , 2 , s 线性无关 )解析:26.设有 3维列向量 (分数:2.00)_正确答案:(正确答案:(1)0 且 -3;(2)=1;(3)=-3)解析:27.已知线性方程组 (分数:2.00)_正确答案:(正确答案:(1)a=1,b=3;(2) 1 =(1,-2,1,0,0) T , 2 =(1,-2,0,1,0) T , 3 =(5,-6,00,1) T ;(3)(-2,3,0,0,0) T +c 1 (1,-2,1,0,0)

27、 T +c 2 (1,-2,01,0) T +c 3 (5,-6,0,0,1) T ,其中 c 1 ,c 2 ,c 3 为任意常数)解析:28.k为何值时,线性方程组 (分数:2.00)_正确答案:(正确答案:(1)当 k-1 且 k4 时,有唯一解: )解析:29.设有线性方程组 (分数:2.00)_正确答案:(正确答案:(1)此时,增广矩阵的行列式是一个 4阶范德蒙行列式,不等于零,故 =4,而 r(A)3故方程组无解;(2)r(A)= )解析:30.设矩阵 A、B 的行数都是 m证明:矩阵方程 AX=B有解的充分必要条件是 r(A)=r(AB)(分数:2.00)_正确答案:(正确答案:设

28、 B、X 按列分块分别为 B=b 1 ,b 2 ,b m ,X=x 1 ,x 2 ,x p ,则 Ax=B即Ax 1 ,Ax 2 ,Ax p =b 1 ,b 2 ,b p ,故 Ax=B有解 线性方程组 Ax j =b j (j=1,2,p)有解,由非齐次线性方程组有解的充要条件,即得 AX=B有解 r(A)=rAb j (j=1,2,p) A的列向量组的极大无关组也是矩阵Ab j (j=1,2,p)的列向量组的极大无关组 )解析:31.设矩阵 (分数:2.00)_正确答案:(正确答案:由下列矩阵的初等行变换: 可见,r(A)=rAB a=1,b=2,c=1,于是由上题知 Ax=B有解 a=1

29、,b=2,c=1此时,对矩阵 D作初等行变换: 于是若将矩阵 B按列分块为 B=b 1 ,b 2 ,b 3 ,则得方程组 Ax=b 1 的通解为: 1 =(1-l,-l,l) T ;方程组 Ax=b 2 的通解为: 2 =(2-m,2-m,m) T ;方程组 Ax=b 3 的通解为: 3 =(1-n,-1-n,n) T ,所以,矩阵方程 Ax=B的通解为 x= 1 , 2 , 3 = )解析:32.已知齐次线性方程组 其中 (分数:2.00)_正确答案:(正确答案:方程组的系数行列式A=b n-1 (b+ a i ),故当A0,即 b0 且 b+ a i 0 时,方程组只有零解当 b=0或 b

30、+ a i =0时,方程组有非零解当 b=0时,设 a 1 0,由系统矩阵 A的初等行变换: 得方程组的基础解系可取为: 当 b+ a i =0时,有 b= a i 0,由系数矩阵的初等行变换: )解析:33.设 A为 n阶方阵(n2),A * 为 A的伴随矩阵,证明: (分数:2.00)_正确答案:(正确答案:当秩(A)=n 时,A * =A n-1 0,故秩(A * )=n当秩(A)=n-1 时,A=0 且 A中至少有某个元素的代数余子式不等于零, A * O, 秩(A * )1,再由 A * A=AE=0 知,A 的列向量均为方程组 A * x=0的解向量, n-秩(A * )秩(A)=

31、n-1, 秩(A * )1,综合前已证过的秩(A * )1,得秩(A * )=1若秩(A)n-2,则 A的每个元素的代数余子式都为零, A * =O, )解析:34.设 1 =(1,2,0) T , 2 =(1,a+2,-3a) T , 3 =(-1,-b-2,a+2b) T ,=(1,3,-3) T ,试讨论当 a,b 为何值时, () 不能由 1 , 2 , 3 线性表示; () 可由 1 , 2 , 3 惟一地线性表示,并求出表示式; () 可由 1 , 2 , 3 线性表示,但表示式不惟一,并求表示式(分数:2.00)_正确答案:(正确答案:设有一组数 x 1 ,x 2 ,x 3 ,使

32、得 x 1 1 +x 2 2 +x 3 3 = (*) 对方程组(*)的增广矩阵施行初等行变换: (1)当 a=0,b 为任意常数时,有 可知 r(A) ,故方程组(*)无解, 不能由 1 , 2 , 3 线性表示 (2)当 a0,且 ab 时,r(A)= =3,方程组(*)有唯一解:x 1 =1- ,x 2 = ,x 3 =0故此时 可由 1 , 2 , 3 唯一地线性表示为:=(1- ) 1 + 2 (3)当 a=b0 时,对 施行初等行变换: 可知 r(A)= =2,故方程组(*)有无穷多解,通解为:x 1 =1- ,x 2 = +c, 3 =c,其中 c为任意常数故此时 可由 1 ,

33、2 , 3 线性表示,但表示式不唯一,其表示式为 =(1- ) 1 +( )解析:35.已知(1,-1,1,-1) T 是线性方程组 (分数:2.00)_正确答案:(正确答案:将解向量 x=(1,-1,1,-1) T 代入方程组,得 =对方程组的增广矩阵施行初等行变换: (1)当 时,有 因 r(A)= =34故方程组有无穷多解,全部解为 x=(0, ,0) T +k(-2,1,-1,2) T ,其中 k为任意常数 因 r(A)= =24,故方程组有无穷多解,全部解为 x=( ,1,0,0) T +k 1 (1,-3,1,0) T +k 2 (-1,-2,0,2) T ,其中 k 1 ,k 2

34、 为任意常数. (2)当 时,由于 x 2 =x 3 ,即 ,故此时,方程组的解为 x= (-2,1,-1,2) T =(-1,0,0,1) T 当 = 时,由于 x 2 =x 3 ,即 1-3k 1 =2k 2 =k 1 ,解得 2 = -2k 1 故此时全部解为 x=( ,1,0,0) T +k 1 (1,-3,1,0) T +( )解析:36.已知齐次线性方程组 (分数:2.00)_正确答案:(正确答案:方程组()的未知量个数大于方程的个数,故方程组()有无穷多个解因为方程组()与()同解,所以方程组()的系数矩阵的秩小于 3由此得 a=2 此时,方程组()的系数矩阵可通过初等行变换化为 由此得(-1,-1,1) T 是方程组()的一个基础解系 将 x 1 =-1,x 2 =-1,x 3 =1代入方程组()可得 b=1,c=2 或 b=0,c=1 当 b=1,x=2 时,对方程组()的系数矩阵施以初等行变换,有比较(1)式与(2)式右边的矩阵可知,此时方程组()与()同解 当 b=0,c=1时,方程组()的系数矩阵可通过初等行变换化为 )解析:

copyright@ 2008-2019 麦多课文库(www.mydoc123.com)网站版权所有
备案/许可证编号:苏ICP备17064731号-1